Page 1 of 1

Putnam 2015/ A3

Posted: Mon Dec 07, 2015 9:51 pm
by Tolaso J Kos
Compute:

\[\log_2\left(\prod_{a=1}^{2015}\prod_{b=1}^{2015}\left(1+e^{2\pi iab/2015}\right)\right)\]
Answer
$13725$
In my opinion, probably the most beautiful question in the competition.